"Bell's theorem refuted" now published by EPL

Foundations of physics and/or philosophy of physics, and in particular, posts on unresolved or controversial issues

Re: "Bell's theorem refuted" now published by EPL

Postby Gordon Watson » Thu Jun 03, 2021 5:28 am

Joy Christian wrote:
Gordon Watson wrote:
If Esail writes A(a, h)= A(a-phi, h) and B(b, h) = B(b-phi -pi/2, h).

Then this is wholly local: for A does not depend on b and B does not depend on a.


But A does depend on b and B does depend on a.


1. NOT in the example that I used above: A(a, h)= A(a-phi, h) and B(b, h) = B(b-phi - pi/2, h).

Joy Christian wrote:
Notice that b' = b - phi - pi/2 implies phi = b - b' - pi/2. Now substitute this in the expression for A, which becomes A(a - b + b' + pi/2, h). And similarly, we can write B in terms of a.


2. Agreed. But in this case, whoever introduces b' [in this way] has needlessly gone beyond my example: A(a, h)= A(a-phi, h) and B(b, h) = B(b - phi-pi/2, h).

3. To be consistent, the introduction of b' should lead to: B(b', h) = B(b' - phi-pi/2, h).

4. Question: Who introduced b' into this discussion of Esail's work? With apologies if I missed it!

PS: To be clear about my position, I am simply seeking to see if Esail begins AND MAINTAINS his study under Einstein-locality. For my part, Bell's theorem is false under freedom of choice, Einstein-locality and non-naive realism.
.
Gordon Watson
 
Posts: 403
Joined: Wed Apr 30, 2014 4:39 am

Re: "Bell's theorem refuted" now published by EPL

Postby Gordon Watson » Thu Jun 03, 2021 5:43 am

Austin Fearnley wrote:Aha. (I seem to be talking to myself on this site ... ?)


Hi Austin,

I know the feeling!

But if you want attention, probably best to start a new thread under something like: Bell's theorem refuted retrocausally.

For my part, Bell's theorem is refuted, without mystery, under freedom of choice, Einstein-locality and non-naive realism.

And certainly without retrocausality.

All the best; Gordon
.
Gordon Watson
 
Posts: 403
Joined: Wed Apr 30, 2014 4:39 am

Re: "Bell's theorem refuted" now published by EPL

Postby FrediFizzx » Thu Jun 03, 2021 6:19 am

Gordon Watson wrote:
Austin Fearnley wrote:Aha. (I seem to be talking to myself on this site ... ?)


Hi Austin,

I know the feeling!

But if you want attention, probably best to start a new thread under something like: Bell's theorem refuted retrocausally.

For my part, Bell's theorem is refuted, without mystery, under freedom of choice, Einstein-locality and non-naive realism.

And certainly without retrocausality.

All the best; Gordon
.

Austin is a guest poster. He can't start a new topic.
.
FrediFizzx
Independent Physics Researcher
 
Posts: 2905
Joined: Tue Mar 19, 2013 7:12 pm
Location: N. California, USA

Re: "Bell's theorem refuted" now published by EPL

Postby Austin Fearnley » Thu Jun 03, 2021 6:37 am

Hi Gordon, and thanks for your comments,

I just wanted information about my interpretation of Esail's method. I have since realised that my first interpretation was wrong despite no-one replying.

I do not want a discussion on this website of my retrocausal method as I have already written at length on a thread started by Richard, and I have papers online.

I side with Richard with respect to Bell's Theorem. Unlike him, though, I am not really interested in the theoretical proof of Bell's Theorem nor in terms like 'realism'. Because of Bell's Theorem one needs a spooky QM or something like retrocausality to find how the Bell Experiment can give the disattenuated QM correlation. I do not like the other main solution 'superdeterminism' as it seems like giving up!

My reason for agreeing with Bell's Theorem is that exact projections give cos theta by definition. Using integer approximations of the exact projections clearly will attenuate the correlation. I do not see how any non-spooky calculations can re-institute the dis-attenuated QM correlation while still using the integer projections. That is all the effort I have ever put into accepting Bell's Theorem theoretically.

I am content with my retrocausal method and am trying to give up physics now. But I had a recent idea that even QM cannot give a generalised proof of the QM Bell correlation. Esail's paper seems to me to endorse that idea, as all measurements of photon 2 are based on photons polarised either for or against alpha of the first detector setting. This is a Malus situation (rather than a generalised situation) for an angle of alpha-beta and corresponds to spooky QM action at a distance. I am not clear why Esail thinks this is local when it is clearly trying to replicate QM spooky interconnections between polarisations or spins of particle pairs.
Austin Fearnley
 

Re: "Bell's theorem refuted" now published by EPL

Postby Joy Christian » Thu Jun 03, 2021 7:07 am

Gordon Watson wrote:
Joy Christian wrote:
Gordon Watson wrote:
If Esail writes A(a, h)= A(a-phi, h) and B(b, h) = B(b-phi -pi/2, h).

Then this is wholly local: for A does not depend on b and B does not depend on a.


But A does depend on b and B does depend on a.


1. NOT in the example that I used above: A(a, h)= A(a-phi, h) and B(b, h) = B(b-phi - pi/2, h).

Joy Christian wrote:
Notice that b' = b - phi - pi/2 implies phi = b - b' - pi/2. Now substitute this in the expression for A, which becomes A(a - b + b' + pi/2, h). And similarly, we can write B in terms of a.


2. Agreed. But in this case, whoever introduces b' [in this way] has needlessly gone beyond my example: A(a, h)= A(a-phi, h) and B(b, h) = B(b - phi-pi/2, h).

3. To be consistent, the introduction of b' should lead to: B(b', h) = B(b' - phi-pi/2, h).

This is mathematically strange. If we write B(b', h) = B(b' - phi-pi/2, h), then that means b' = b' - phi-pi/2, which imples phi + pi/2 = 0 always. So what is the point of it, mathematically?

There was a very good reason why Esail was very reluctant to write down mathematical functions A(a, h) and B(b, h) until I pushed him. They reveal some strangeness of the model.
.
Joy Christian
Research Physicist
 
Posts: 2793
Joined: Wed Feb 05, 2014 4:49 am
Location: Oxford, United Kingdom

Re: "Bell's theorem refuted" now published by EPL

Postby minkwe » Thu Jun 03, 2021 7:36 am

Austin Fearnley wrote:Aha. (I seem to be talking to myself on this site ... ?)

Up to now I had assumed that the 'predicting' paragraphs of Esail's paper were merely post-experiment calculations but now I realise they are not.

As Esail's paper is a forwards-in-time model then photon 2 gets a very spooky (normal QM spookiness?) alteration of its polarisation setting after photon 1 is measured but before photon 2 is itself measured. That is why alpha (or alpha - pi/2 appears in the calculation of B (rather than beta) for photon 2. This of course means that the B measurement (at setting beta) is always carried out on a particle polarised along or against alpha. This will give the required Bell correlation of 0.7070 for alpha-beta =22.5 degrees. But this method uses spooky (QM type of spooky) action at a distance. Using the Malus Law variable of cos^2 (delta) ensures the 0.707 correlation as Alice is acting like the first polarisation filter in a Malus experiment while Bob is acting like the second filter in a Malus exeriment. The cos^2 probability ensures 85% of photons pass the second filter at 22.5 degrees, which is always equivalent to a correlation of 0.707.

Remember that Quantum "Mechanics" gives you correlations without any mechanics of how they arise. Secondly, the EPR experiment is exactly analogous to a Malus experiment with one arm of the experiment acting as the filter for the other. This is exactly how EPR experiments are done in practice. Unfortunately, many theoreticians do not appreciate this fact. Experimenters spend their time looking for ingenuous ways of filtering results on one arm based on information from the other. All the care that is done during the actual measurements to ensure that information does not travel from one side to the other is completely irrelevant. Everything happens at the selection step. Nowadays, they try to build in the selection into the measurements and they call it "heralding". But it's all the same trick. It's like magic, to see the trick, you have to ignore the mountains of subterfuge.
minkwe
 
Posts: 1441
Joined: Sat Feb 08, 2014 10:22 am

Re: "Bell's theorem refuted" now published by EPL

Postby Esail » Thu Jun 03, 2021 7:46 am

Gordon Watson wrote:
If Esail writes A(a, h)= A(a-phi, h) and B(b, h) = B(b-phi -pi/2, h).

Then this is wholly local: for A does not depend on b and B does not depend on a.


I should have written

A'(a, h)= A(a-phi, h) and B'(b, h) = B(b-phi -pi/2, h).
as A and A' as well as B and B' are different functions as they have different arguments although they denote each both the measurement result.
Esail
 
Posts: 132
Joined: Sun Jan 28, 2018 10:44 am

Re: "Bell's theorem refuted" now published by EPL

Postby Joy Christian » Thu Jun 03, 2021 8:01 am

Esail wrote:
Gordon Watson wrote:
If Esail writes A(a, h)= A(a-phi, h) and B(b, h) = B(b-phi -pi/2, h).

Then this is wholly local: for A does not depend on b and B does not depend on a.


I should have written

A'(a, h)= A(a-phi, h) and B'(b, h) = B(b-phi -pi/2, h).
as A and A' as well as B and B' are different functions as they have different arguments although they denote each both the measurement result.

That does not change anything. Hiding the phi-dependence from A' and B' does not change the fact that it is there and makes the model either superdeterministic, retrocausal, or nonlocal.
.
Joy Christian
Research Physicist
 
Posts: 2793
Joined: Wed Feb 05, 2014 4:49 am
Location: Oxford, United Kingdom

Re: "Bell's theorem refuted" now published by EPL

Postby Austin Fearnley » Thu Jun 03, 2021 1:30 pm

Hi Minkwe
Minkwe wrote:
Remember that Quantum "Mechanics" gives you correlations without any mechanics of how they arise.

Yes, and I am wondering if the QM mechanics could be retrocausal.
Secondly, the EPR experiment is exactly analogous to a Malus experiment with one arm of the experiment acting as the filter for the other. This is exactly how EPR experiments are done in practice.

Well, my particle model allows a classical determination of Malus's Law. A cheat really as I reverse engineer my model from Malus's Law by differentiating cos^2. But I cannot get the Bell correlation from Malus without retrocausality. Malus means measuring one electron twice, in succession. So nothing spooky in the spins after successive measurements. Bell involves two measurements on different (entangled) particles and the entangled state must be engineered or maintained somehow. Spookily with QM and non-spookily with retrocausality. I imagine retrocausality will mean 'no quantum computers'.
Unfortunately, many theoreticians do not appreciate this fact. Experimenters spend their time looking for ingenuous ways of filtering results on one arm based on information from the other. All the care that is done during the actual measurements to ensure that information does not travel from one side to the other is completely irrelevant. Everything happens at the selection step. Nowadays, they try to build in the selection into the measurements and they call it "heralding". But it's all the same trick. It's like magic, to see the trick, you have to ignore the mountains of subterfuge.

Thanks. I looked at a 2019 paper on 'heralding' by Zhao et al but found it far too strange for me to learn anything. The practical side of Bell is a mystery to me.
Austin Fearnley
 

Re: "Bell's theorem refuted" now published by EPL

Postby FrediFizzx » Thu Jun 03, 2021 2:37 pm

Joy Christian wrote:
Gordon Watson wrote:
If Esail writes A(a, h)= A(a-phi, h) and B(b, h) = B(b-phi -pi/2, h).

Then this is wholly local: for A does not depend on b and B does not depend on a.


But A does depend on b and B does depend on a.

Notice that b' = b - phi - pi/2 implies phi = b - b' - pi/2. Now substitute this in the expression for A, which becomes A(a - b + b' + pi/2, h). And similarly, we can write B in terms of a.
.

Not only that but his functions are absurd. They don't even produce +/-1's. Esail, write out your A and B functions completely so they produce +/-1's.
.
FrediFizzx
Independent Physics Researcher
 
Posts: 2905
Joined: Tue Mar 19, 2013 7:12 pm
Location: N. California, USA

Re: "Bell's theorem refuted" now published by EPL

Postby Joy Christian » Thu Jun 03, 2021 3:01 pm

FrediFizzx wrote:
Joy Christian wrote:
Gordon Watson wrote:
If Esail writes A(a, h)= A(a-phi, h) and B(b, h) = B(b-phi -pi/2, h).

Then this is wholly local: for A does not depend on b and B does not depend on a.


But A does depend on b and B does depend on a.

Notice that b' = b - phi - pi/2 implies phi = b - b' - pi/2. Now substitute this in the expression for A, which becomes A(a - b + b' + pi/2, h). And similarly, we can write B in terms of a.
.

Not only that but his functions are absurd. They don't even produce +/-1's. Esail, write out your A and B functions completely so they produce +/-1's.

Oh ... In that case, it is not even a model of a Bell-test experiment, let alone a local model. How on earth such a paper passed peer review?
.
Joy Christian
Research Physicist
 
Posts: 2793
Joined: Wed Feb 05, 2014 4:49 am
Location: Oxford, United Kingdom

Re: "Bell's theorem refuted" now published by EPL

Postby Gordon Watson » Thu Jun 03, 2021 3:28 pm

Joy Christian wrote:
Gordon Watson wrote:
Joy Christian wrote:
Gordon Watson wrote:
If Esail writes A(a, h)= A(a-phi, h) and B(b, h) = B(b-phi -pi/2, h).

Then this is wholly local: for A does not depend on b and B does not depend on a.


But A does depend on b and B does depend on a.


1. NOT in the example that I used above: A(a, h)= A(a-phi, h) and B(b, h) = B(b-phi - pi/2, h).

Joy Christian wrote:
Notice that b' = b - phi - pi/2 implies phi = b - b' - pi/2. Now substitute this in the expression for A, which becomes A(a - b + b' + pi/2, h). And similarly, we can write B in terms of a.


2. Agreed. But in this case, whoever introduces b' [in this way] has needlessly gone beyond my example: A(a, h)= A(a-phi, h) and B(b, h) = B(b - phi-pi/2, h).

3. To be consistent, the introduction of b' should lead to: B(b', h) = B(b' - phi-pi/2, h).

This is mathematically strange. If we write B(b', h) = B(b' - phi-pi/2, h), then that means b' = b' - phi-pi/2, which imples phi + pi/2 = 0 always. So what is the point of it, mathematically?

There was a very good reason why Esail was very reluctant to write down mathematical functions A(a, h) and B(b, h) until I pushed him. They reveal some strangeness of the model.
.


1. I was simply making a mathematical point for Esail, etc, bound (as we all are) by Einstein-locality:

Given B(b', h) = B(b' - phi - pi/2, h), then one possibility is cos(x - h) = cos(x + pi/2 - pi/2 - h) when phi = -pi/2, -5pi/2, etc:

when I was still wondering about the b'. [For equivalence relations (like I use), might still save him!]

Alas: it's evidently a typo; see next.


Esail wrote:
Gordon Watson wrote:
If Esail writes A(a, h)= A(a-phi, h) and B(b, h) = B(b-phi -pi/2, h).

Then this is wholly local: for A does not depend on b and B does not depend on a.


I should have written

A'(a, h)= A(a-phi, h) and B'(b, h) = B(b-phi -pi/2, h).
as A and A' as well as B and B' are different functions as they have different arguments although they denote each both the measurement result.


I happily leave Esail to explain the point of his strangeness.
.
Gordon Watson
 
Posts: 403
Joined: Wed Apr 30, 2014 4:39 am

Re: "Bell's theorem refuted" now published by EPL

Postby gill1109 » Thu Jun 03, 2021 9:38 pm

Joy Christian wrote:
FrediFizzx wrote:
Joy Christian wrote:
Gordon Watson wrote:
If Esail writes A(a, h)= A(a-phi, h) and B(b, h) = B(b-phi -pi/2, h).

Then this is wholly local: for A does not depend on b and B does not depend on a.


But A does depend on b and B does depend on a.

Notice that b' = b - phi - pi/2 implies phi = b - b' - pi/2. Now substitute this in the expression for A, which becomes A(a - b + b' + pi/2, h). And similarly, we can write B in terms of a.
.

Not only that but his functions are absurd. They don't even produce +/-1's. Esail, write out your A and B functions completely so they produce +/-1's.

Oh ... In that case, it is not even a model of a Bell-test experiment, let alone a local model. How on earth such a paper passed peer review?
.

Yeah! I agree with Joy, Fred and Gordon!

How did the paper pass peer review? Peer review is nowadays much too fast and superficial and taken over by AI bots, which pick referees and just count positive and negative reviews. Papers are published with more than 30% positive reviews. Editors with subject matter knowledge are almost entirely taken out of the loop. They are far too busy writing their own papers and research proposals and adding their names to the papers written by their students and cultivating their CV’s with more and more editorships of prestigious journals.
gill1109
Mathematical Statistician
 
Posts: 2812
Joined: Tue Feb 04, 2014 10:39 pm
Location: Leiden

Re: "Bell's theorem refuted" now published by EPL

Postby Joy Christian » Thu Jun 03, 2021 10:18 pm

gill1109 wrote:
How did the paper pass peer review? Peer review is nowadays much too fast and superficial and taken over by AI bots, which pick referees and just count positive and negative reviews. Papers are published with more than 30% positive reviews. Editors with subject matter knowledge are almost entirely taken out of the loop. They are far too busy writing their own papers and research proposals and adding their names to the papers written by their students and cultivating their CV’s with more and more editorships of prestigious journals.

The publication of your paper in the journal Entropy is a prime example of the deterioration in the peer review system. Your paper contained at least one extremely elementary mathematical mistake and other unacceptable content, which you have now admitted publicly: https://www.mdpi.com/1099-4300/23/5/631

Image

And yet such a paper was published by Entropy. And even the post-publication reviewer was not able to detect your mistakes. So how on earth did your paper passed peer reviews?
.
Joy Christian
Research Physicist
 
Posts: 2793
Joined: Wed Feb 05, 2014 4:49 am
Location: Oxford, United Kingdom

Re: "Bell's theorem refuted" now published by EPL

Postby Austin Fearnley » Fri Jun 04, 2021 5:16 am

Hi all

Bell's Theorem is not disproved; and the method is non-local.

But despite that I enjoyed Esail's paper when I finally understood it more.
It is IMO very similar to my retrocausal paper except where I use non-spooky retrocausality, Esail uses spooky QM action at a distance. I found his paper useful in endorsing my feelings that QM cannot cope with a non-Malus arrangement of particle polarisation vectors.

I could make a computer program of Esail's model, and it would give the spooky correlation. I initially made an Excel spreadsheet of Esail's model as I originally and erroneously thought the model must be. As I understood it to be local I used alpha in the calculation of A and used beta in the calculation of B. That gave a correl of 0.37 to 0.5 (some approximations caused this range; and, ignore sign of correlation) instead of 0.707. These correlations were similar to my correlations when I tried to use Malus without rerocausality in my model. Correlations can be lower than 0.5 because the probability aspect lowers the correlation even more than a fixed vector approach. With retrocausality my model gives the 0.707 correlation.

But Esail uses alpha in both A and B calculations. Esail using alpha for B is QM spooky and non-local. My method uses alpha for B but via non-spooky retrocausality, and local effects.

So what is spooky or non-spooky here? IMO a particle polarisation should stay unchanged until it has a measurement event. The polarisation of a measured particle (photon 1) should change on exiting the detector after measurement. Esail, IMO, allows the polarisation of photon 2 to change spookily before photon 2 is measured, though after photon 1 is measured. Presumably this change relies on spooky entanglement. I say that my retrocausal model is non-spooky and local because all polarisation changes occur immediately after a measurement (but 'after' is in the time direction of the appropriate particle).

Anyway, I knew that I could program Esail's model so I went back to the Excel spreadsheet to amend it to have alpha used in the B calculations. However, it now seemed so trivial that all I needed to do was write the answer = 0.707 derived exactly fom a Malus 85 % intensity at alpha-beta = 22.5 degrees for photons. But it could be bulked out more, particle-at-a-time if required either in Excel or in VB, though the overall Malus calculation is trivial. Not worth the coding.

A few more points.

Esail's paper gives probabilities of A and B outcomes. That is fine for me and does not mean that As and Bs cannot be calculated. It just means that some random numbers need generating for individual particle outcomes.

The outcomes A and B depend on the order of measurement so that 'A then B' is not the same outcome as 'B then A'. This is fine as the model is probabilistic and in any case need not respect counterfactual definiteness (which, further, would cause it to fail the Quantum Randi challenge if it did respect counterfactual definiteness.)

Why does Esail think that this is a local method? I suspect that it is something to do with interpretations of what is going on wih polarisation effects at the detectors. IMO what happens at the detectors is not spooky and cannot be used to insert the spooky QM entanglement effect via a side door. The method is clearly non-local but I like the paper anyway.
Austin Fearnley
 

Re: "Bell's theorem refuted" now published by EPL

Postby gill1109 » Fri Jun 04, 2021 1:00 pm

Austin Fearnley wrote:Hi all

Bell's Theorem is not disproved; and the method is non-local.

But despite that I enjoyed Esail's paper when I finally understood it more.
It is IMO very similar to my retrocausal paper except where I use non-spooky retrocausality, Esail uses spooky QM action at a distance. I found his paper useful in endorsing my feelings that QM cannot cope with a non-Malus arrangement of particle polarisation vectors.



A few more points.

Esail's paper gives probabilities of A and B outcomes. That is fine for me and does not mean that As and Bs cannot be calculated. It just means that some random numbers need generating for individual particle outcomes.

The outcomes A and B depend on the order of measurement so that 'A then B' is not the same outcome as 'B then A'. This is fine as the model is probabilistic and in any case need not respect counterfactual definiteness (which, further, would cause it to fail the Quantum Randi challenge if it did respect counterfactual definiteness.)

Why does Esail think that this is a local method? I suspect that it is something to do with interpretations of what is going on wih polarisation effects at the detectors. IMO what happens at the detectors is not spooky and cannot be used to insert the spooky QM entanglement effect via a side door. The method is clearly non-local but I like the paper anyway.

Nice comments! Yet more agreement. This analysis helps me understand Esail’s approach better.

Personally I find retrocausality spooky, but anyway, we need something non-local to create those “spooky” quantum correlations.
gill1109
Mathematical Statistician
 
Posts: 2812
Joined: Tue Feb 04, 2014 10:39 pm
Location: Leiden

Re: "Bell's theorem refuted" now published by EPL

Postby gill1109 » Fri Jun 04, 2021 1:15 pm

Joy Christian wrote:
gill1109 wrote:
How did the paper pass peer review? Peer review is nowadays much too fast and superficial and taken over by AI bots, which pick referees and just count positive and negative reviews. Papers are published with more than 30% positive reviews. Editors with subject matter knowledge are almost entirely taken out of the loop. They are far too busy writing their own papers and research proposals and adding their names to the papers written by their students and cultivating their CV’s with more and more editorships of prestigious journals.

The publication of your paper in the journal Entropy is a prime example of the deterioration in the peer review system. Your paper contained at least one extremely elementary mathematical mistake and other unacceptable content, which you have now admitted publicly: https://www.mdpi.com/1099-4300/23/5/631

Image

And yet such a paper was published by Entropy. And even the post-publication reviewer was not able to detect your mistakes. So how on earth did your paper passed peer reviews?
.

How on Earth, indeed? At the review of my corrected version, half a dozen highly skilled and highly respected scientists detected no mistakes! Both mathematicians and physicists. They took quite a while over it, because they wanted to be quite sure. Joy’s complaint against me was very serious and was, correctly, taken very seriously. They had access to Joy’s critique of my paper.

(The mathematical error I had made in the originally published version was a trivial and inconsequential error; I had noticed it, and asked for a correction note myself, very soon after the initial publication. I was annoyed it took so long to put out the correction - that delay was because of Joy’s complaint).

How could that be, indeed?
gill1109
Mathematical Statistician
 
Posts: 2812
Joined: Tue Feb 04, 2014 10:39 pm
Location: Leiden

Re: "Bell's theorem refuted" now published by EPL

Postby Joy Christian » Fri Jun 04, 2021 1:34 pm

gill1109 wrote:
At the review of my corrected version, half a dozen highly skilled and highly respected scientists detected no mistakes! Both mathematicians and physicists.

Those "highly skilled and highly respected scientists ... both mathematicians and physicists" are very lucky that they are anonymous. For, in due course, I will be exposing their extraordinary incompetence, by exposing the mistakes in the corrected version of your paper, line by line. I will be doing that at a time and venue of my choosing. So get ready for more correction notes.
.
Joy Christian
Research Physicist
 
Posts: 2793
Joined: Wed Feb 05, 2014 4:49 am
Location: Oxford, United Kingdom

Re: "Bell's theorem refuted" now published by EPL

Postby Joy Christian » Fri Jun 04, 2021 11:25 pm

Austin Fearnley wrote:
Bell's Theorem is not disproved...

Bell's theorem did not require disproving. To begin with, Bell's theorem was a nonstarter. Moreover, I disproved it on the 20th of March 2007 by demonstrating that the measurement results, +/-1, observed by Alice and Bob are limiting scalar points of a quaternionic 3-sphere, taken as a model of the 3D physical space. Thus, Bell's theorem has been dead for more than 14 years.

In Section II of my latest paper (open access), I explain why Bell's theorem was a nonstarter, and in Section III of the same paper, I summarize this 3-sphere model of the singlet correlations.
.
Joy Christian
Research Physicist
 
Posts: 2793
Joined: Wed Feb 05, 2014 4:49 am
Location: Oxford, United Kingdom

Re: "Bell's theorem refuted" now published by EPL

Postby Austin Fearnley » Sat Jun 05, 2021 2:17 am

Hi Joy

Your comment assumes that everone agrees with you that you have already killed off Bell's Theorem.

I am just an amateur physicist, starting ten to fifteen years ago on Susskind's 'Theoretical Minimum' online courses, after I retired from full-time work. I had studied physics though as a subsid at university. I do not have the time (at my age) nor energy nor ability nor inclination to read all your papers thoroughly though I have skim read most of them. I adhere to the theoretical minimum strategy as much as possible. I looked at your two papers that you indicated and they are still too long for me to want to read thoroughly. Fred's website has been very useful though to pick up ideas on your model.

I did read your one-page paper carefully and never fully accepted it. I do not see the point of reading higher dimensional versions of it when I see doubts in the S3 version.

Some general comments. It is strange how some comments stay in one's head. Years ago on the old version of s.p.f. in a discussion of your one-page paper (I think) Jos wrote along the lines that if S3 is locally flat then why should a curvature of the universe affect measurements in a lab. I hope I remember this correctly as I could probably not find that thread again now. I still have never seen that comment resolved.

I like your ideas, of course, and especially the geometry of S3. As I understand it, you would accept Bell's Theorem to be true if the universe were to be completely R3. And my comments were assuming R3 applied. I am still working on my own model and I like the idea of the universe and of particles all being S3.

We had a swap of ideas years ago and I suggested that the hidden variables should be + for one particle and - for the other. Your model uses + for both particles in one pair or - for both particles in one pair.

Going back to Jos's comment, if your + and - refer to the universe's curvature, this assumes that the universe is curving both ways at the same time. (While being locally flat in each case.) This seems to me to a little like having the universe be dS and AdS at the same time.

My model is still in progress but I am linking the unverse's arrow of time to the curvature of the background space (after Chappell et al). So when you assume (say) + for the universe, time would be following its normal course, but for - time would run backwards. Remember your analogy of skaters skating 'mirror image backwards' at the far end of the Moibus loop. Well, are they skating 'mirror image backwards' in space or is their time running backwards.

The all-important point at infinity in S3 closes the geometry. A realistic model needs to incorporate time. Does the closure at infinity close time too. In which case, we should not be surprised to meet a dancer (positron) travelling backwards in time through our labs. So my model would have + for electron and - for positron within the same pair. This would IMO get around Jos's objection. These +s and -s would be set in the universes overall + curvature.

But can we access the opposite curvature of the universe starting from here. Penrose's CCC model would seem to deny it as our time direction would cease to exist before accessing the spacetime of the opposite curvature.


Sorry. Probably not the sort of comments you were expecting...
Austin Fearnley
 

PreviousNext

Return to Sci.Physics.Foundations

Who is online

Users browsing this forum: ahrefs [Bot] and 76 guests

cron
CodeCogs - An Open Source Scientific Library